The perimeter of a rectangle is 18cm . if the length is (x+2), find it's width.​

Answers

Answer 1

Answer:

W = 7 - x

Step-by-step explanation:

The perimeter is P= 2L + 2×W , where L is the length and W is the width.

If L = (x+2) , replacing L with the expression x+2 we have

P= 2×(X+2) + 2W ⇔ 18 = 2x + 4 + 2W ⇔ 2W =18 - 2x - 4 ⇔ 2W = 14 - 2x

⇔ W = 7 - x


Related Questions


(

2
,
2
)
(−2,2) under a translation is
(

3
,
5
)
(−3,5). Find the coordinates of the image of the point
(

3
,
1
)
(−3,1) under the same translation.

Answers

Answer:

kslskd बोडलालामसूनिविवल

CAN SOMEONE PLS TELL ME DOMAIN AND RANGE

Answers

Answers:

Domain is (-4, 3]Range is (-5, 5]

=====================================================

Explanation:

The domain is the set of allowed x input values, aka the set of all allowed x coordinates of the points. We see that [tex]-4 < x \le 3[/tex]. It might help to draw vertical lines through the endpoints until you reach the x axis. Note the open hole at x = -4 to indicate we do not include this as part of the domain (hence the lack of "or equal to" for the first inequality sign).

The interval [tex]-4 < x \le 3[/tex] then can be condensed into the shorthand form (-4, 3] which is the domain in interval notation.

It says: x is between -4 and 3. It can't equal -4 but it can equal 3.

So the use of parenthesis versus square brackets tells the reader which endpoint is included or not.

--------------------------------------------------

The range describes all possible y outputs. We see that y = 5 is the largest it gets and y = -5 is the lower bound. It might help to draw horizontal lines through the endpoints until you reach the y axis. The open hole means -5 is not part of the range.

The range as a compound inequality is [tex]-5 < y \le 5[/tex]. This condenses into the shorthand of (-5, 5] which is the range in interval notation.

Verbally, the range is the set of y values such that y is between -5 and 5. It can't equal -5 but it can equal 5.

Answer:

Hello,

Step-by-step explanation:

Domain(f)= ]-4;3]={x€R | -4<x≤3}

Rang(f)=]-5;5]={y€R | -5< y ≤5}

A hemispherical tank is filled with water and has a diameter of 16 feet. If water weighs 62.4 pounds per cubic foot, what is the total weight of the water in a full tank, to the nearest pound?

Answers

Answer: 133,827 lb/ft^3

Step-by-step explanation:

V = (4/3) (3.14)(8^3) = 133,827

Can u help solve this

Answers

Answer:

2

Step-by-step explanation:

slope = rise/run

rise = difference in y

run = difference in x

slope = (8 - 2)/(3 - 0)

slope = 6/3

slope = 2

Answer:

Slope = 2

Step-by-step explanation:

Use the Slope-Formula to find the slope

m = slope

m = change in y/change in x

m = (y2 - y1)/(x2 - x1)

Points: (0, 2) (3, 8) where the first point is x1 & y1 and the second point is x2 & y2.

m = (8 - 2)/(3 - 0)

m = 6/3

m = 2

The slope will be 2

- 100
-V4
L.A.1
ints
3. What is the simplest form of
a. -51
b. 25i
c. -251
d. 5i

Answers

Answer:

- 5i

Step-by-step explanation:

sqrt(-100) / -sqrt(4)

We know that  sqrt(a) / sqrt(b) = sqrt(a/b)

- sqrt(-100/4)

- sqrt(-25)

We know that sqrt(ab) = sqrt(a) sqrt(b)

- sqrt(25) sqrt(-1)

We know sqrt(-1) = i

Taking the principle value of sqrt(25)

- 5i

for what value of x does 4^x=(1/8)^x+5?

Answers

Use main properties of powers

(a^m)^n=a^{m\cdot n};(am)n=am⋅n;

\dfrac{1}{a^n}=a^{-n}an1=a−n

to simplify given equation.

1.

4^x=(2^2)^x=2^{2x}.4x=(22)x=22x.

2.

\left(\dfrac{1}{8}\right)^{x+5}=\left(\dfrac{1}{2^3}\right)^{x+5}=(2^{-3})^{x+5}=2^{-3x-15}.(81)x+5=(231)x+5=(2−3)x+5=2−3x−15.

3. Then the equation is

2^{2x}=2^{-3x-15}.22x=2−3x−15.

The bases are the same, so equate the powers:

2x=-3x-15,

2x+3x=-15,

5x=-15,

x=-3.

Answer: for x=-3.

3x (x - 2) = x^2 - 4

Answers

Answer:

x = 1, x = 2

Step-by-step explanation:

Given

3x(x - 2) = x² - 4 ← distribute parenthesis on left side

3x² - 6x = x² - 4 ( subtract x² - 4 from both sides )

2x² - 6x + 4 = 0 ( divide through by 2

x² - 3x + 2 = 0 ← in standard form

Consider the factors of the constant term (+ 2) which sum to give the coefficient of the x- term (- 3)

The factors are - 1 and - 2 , since

- 1 × - 2 = + 2 and - 1 - 2 = - 3 , then

(x - 1)(x - 2) = 0 ← in factored form

Equate each factor to zero and solve for x

x - 1 = 0 ⇒ x = 1

x - 2 = 0 ⇒ x = 2

Hey guys ♡
hw r u all doin
pls solve this question for me ⤵️

(m-1)/2 - (m-2)/3 = ( m - 4 )/7 ​

Answers

Answer:

m = - 31

Step-by-step explanation:

Given

[tex]\frac{m-1}{2}[/tex] - [tex]\frac{m-2}{3}[/tex] = [tex]\frac{m-4}{7}[/tex]

Multiply through by 42 , the LCM of 2, 3 and 7 , to clear the fractions

21(m - 1) - 14(m - 2) = 6(m - 4) ← distribute parenthesis

21m - 21 - 14m + 28 = 6m - 24

7m + 7 = 6m - 24 ( subtract 6m from both sides )

m + 7 = - 24 ( subtract 7 from both sides )

m = - 31

When simplified and written in standard form, which quadratic function is equivalent to the polynomial shown?

Answers

Answer:

The Polynominal is not shown. So how can i answer???

Step-by-step explanation:

Thank You! Hope it helps! Please Mark me brainliest!

Use the function below to find F(-4).
F(x) = 2^x

Answers

Answer:

Step-by-step explanation:

F(x)=2^x

[tex]F(-2)=2^{-2} =\frac{1}{2^2} =\frac{1}{4}[/tex]

So it’s pretty much how the other person did it, except you substitute -4 for x. That will give you 2^-4 which is 1/(2^4)= 1/16

Simplify the complex number √-16+8.

Answers

Answer:

[tex]\sqrt{-16} +8[/tex]

[tex]=4i+8[/tex]

OAmalOHopeO

answer please it's urgent​

Answers

Answer:

HOPE IT HELP YOU . Plz tell me if it is write or wrong .

Please help I don’t understand math

Answers

Answer:

x = 12

Step-by-step explanation:

The sum of the two smaller angles is 140

80+5x = 140

Subtract 80 from each side

80+5x-80 =140-80

5x = 60

Divide each side by 5

5x/5 = 60/5

x = 12

Find X
Round to the nearest tenth

Answers

Answer:

x=437.3 ft

Step-by-step explanation:

cos(29)=B/H, cos(29)=x/500. x=437.3 ft

Help ! With step by step solution

Answers

Answer:

the correct answer is C

Step-by-step explanation:

first step you change the x and y powers to the square root(showing in the pic).

second the number outside of the parenthesis times the numbers inside the parenthesis and then you just figure it out how it works...

I need help plss ( math ) SOMEONE ASAP

Answers

26a)729

b)2,457.6

c)3,500

d)8,100

e) 89,600

Step-by-step explanation:

just multiply first and divide with upper number to the lower

Answer it please ↓↓

Fubuki drives 18.4 miles in 16 minutes. He passes a sign which gives the speed limit as 60 mph. By how much, in mph, did Fubuki's average speed exceed the speed limit?

Answers

Step-by-step explanation:

hope this helps to u......

Which expression is equivalent to −24x+72?

−24(x−72)negative 24 times open paren x minus 72 close paren

−24(x−3)negative 24 times open paren x minus 3 close paren

24(x+3)24 times open paren x plus 3 close paren

24(x+72)

Answers

Answer:-24(x-3) = -24x (-24)(-3)

Step-by-step explanation:

Answer:

eat my hand where the pic

Step-by-step explanation:

yeah

Rewrite in simplest radical form the problem in the image below. Show and explain each step. Thank you for your time and help.

Answers

Answer:

Hello,

Step-by-step explanation:

[tex]\dfrac{x^{\frac{5}{6}} }{x^{\frac{1}{6}} } \\\\=x^{\frac{5}{6} -\frac{1}{6} }\\\\=x^{\frac{4}{6} }\\\\=x^{\frac{2}{3} }\\\\=\sqrt[3]{x^2}[/tex]

Hi there!

Look the picture for your answer.

Hope it helps!

Somebody can help me please

Answers

It’s 25. Plug -2 into g, which is -5. Then plug -5 into f, which is 25.

Write the fraction or mixed number as a decimal.

Answers

Vas happenin!!!
1.725
Hope this helps *smiles*
Agreed with guy above

What is the length of AC?

Answers

Because all the angles are congruent (the same), this is an equilateral triangle. All equilateral triangles have congruent angles and congruent sides, so all sides has to be 14.

Need help asapppppppppp

Answers

The answer is 168 degrees

If sin A= 0.8, find the positive value of cos A​

Answers

Answer:

cosA = 0.6

Step-by-step explanation:

Using the Pythagorean identity

sin²A + cos²A = 1 ( subtract sin²A from both sides )

cos²A = 1 - sin²A ( take the square root of both sides )

cosA = ± [tex]\sqrt{1-sin^2A[/tex]

Since only the positive value is required , then

cosA = [tex]\sqrt{1-(0.8)^2}[/tex]

         = [tex]\sqrt{1-0.64}[/tex]

         = [tex]\sqrt{0.36}[/tex]

         = 0.6

Answer:

Answer:

Answer:x = 25°

Answer:x = 25°Step-by-step explanation:

Answer:x = 25°Step-by-step explanation:The secant- secant angle FGE is one half the difference of the intercepted arcs, that is

Answer:x = 25°Step-by-step explanation:The secant- secant angle FGE is one half the difference of the intercepted arcs, that is\frac{1}{2}

Answer:x = 25°Step-by-step explanation:The secant- secant angle FGE is one half the difference of the intercepted arcs, that is\frac{1}{2} 2

Answer:x = 25°Step-by-step explanation:The secant- secant angle FGE is one half the difference of the intercepted arcs, that is\frac{1}{2} 21

Answer:x = 25°Step-by-step explanation:The secant- secant angle FGE is one half the difference of the intercepted arcs, that is\frac{1}{2} 21

Answer:x = 25°Step-by-step explanation:The secant- secant angle FGE is one half the difference of the intercepted arcs, that is\frac{1}{2} 21 ( BD - FE ) = ∠ FGE, that is

Answer:x = 25°Step-by-step explanation:The secant- secant angle FGE is one half the difference of the intercepted arcs, that is\frac{1}{2} 21 ( BD - FE ) = ∠ FGE, that is\frac{1}{2}

Answer:x = 25°Step-by-step explanation:The secant- secant angle FGE is one half the difference of the intercepted arcs, that is\frac{1}{2} 21 ( BD - FE ) = ∠ FGE, that is\frac{1}{2} 2

Answer:x = 25°Step-by-step explanation:The secant- secant angle FGE is one half the difference of the intercepted arcs, that is\frac{1}{2} 21 ( BD - FE ) = ∠ FGE, that is\frac{1}{2} 21

Answer:x = 25°Step-by-step explanation:The secant- secant angle FGE is one half the difference of the intercepted arcs, that is\frac{1}{2} 21 ( BD - FE ) = ∠ FGE, that is\frac{1}{2} 21

Answer:x = 25°Step-by-step explanation:The secant- secant angle FGE is one half the difference of the intercepted arcs, that is\frac{1}{2} 21 ( BD - FE ) = ∠ FGE, that is\frac{1}{2} 21 (135 - x) = 55 ( multiply both sides by 2 to clear the fraction )

Answer:x = 25°Step-by-step explanation:The secant- secant angle FGE is one half the difference of the intercepted arcs, that is\frac{1}{2} 21 ( BD - FE ) = ∠ FGE, that is\frac{1}{2} 21 (135 - x) = 55 ( multiply both sides by 2 to clear the fraction )135 - x = 110 ( subtract 135 from both sides )

Answer:x = 25°Step-by-step explanation:The secant- secant angle FGE is one half the difference of the intercepted arcs, that is\frac{1}{2} 21 ( BD - FE ) = ∠ FGE, that is\frac{1}{2} 21 (135 - x) = 55 ( multiply both sides by 2 to clear the fraction )135 - x = 110 ( subtract 135 from both sides )- x = - 25 ( multiply both sides by - 1 )

Answer:x = 25°Step-by-step explanation:The secant- secant angle FGE is one half the difference of the intercepted arcs, that is\frac{1}{2} 21 ( BD - FE ) = ∠ FGE, that is\frac{1}{2} 21 (135 - x) = 55 ( multiply both sides by 2 to clear the fraction )135 - x = 110 ( subtract 135 from both sides )- x = - 25 ( multiply both sides by - 1 )x = 25°

The graph of h(x) = (x - 3)2 is a translation of the
graph of f(x) ….. blank
by
…. Blank units.

Answers

Answer:

right by 3 I think

Step-by-step explanation:

Answer: Right by 3 Units

Step-by-step explanation:

Right on edge 2021

In ΔRST, m∠R = 92° and m∠S = 71°. Which list has the sides of ΔRST in order from shortest to longest?

Answers

Answer:

RS, RT, ST

Step-by-step explanation:

We require the third angle in the triangle

∠ T = 180° - (92 + 71)° = 180° - 163° = 17°

The shortest side is opposite the smallest angle

∠ T = 17° → opposite side RS

The longest side is opposite the largest angle

∠ R = 92° → opposite side ST

Then sides from shortest to longest is

RS, RT, ST

……………….pls and thx——————-

Answers

Answer:

14 yards shorter

Step-by-step explanation:

Use Pythagoras' Theorem

a²+b²=c²

16²+63²=4225

√4225= 65yd

The diagonal line (c) is 65 yards long

16 + 63 = 79 yd

It would be 14 yards shorter (79-65)

what is the answer ,and how do you solve it .please help

Answers

Answer:

10

Step-by-step explanation:

2000/ 2x = 10

Multiply each side by 2x

2000/ 2x = 10 *2x

2000 = 20x

Divide by 20

2000/20 = 20x/20

100 = x

Take the square root of each side

sqrt(100) = sqrt(x)

10 = sqrt(x)

A train travelling at 30km/hour reaches a tunnel which is 9 times as long as the train. If the train takes 2 minutes to completely clear the tunnel, how long is the train?​

Answers

Answer: 111.1 m

Step-by-step explanation:

(30*2)/60 = 1 km ( the tunnel length is 1 Km.)

1 Km = 1000 m.

1000/9 = 111.1 m.

Is it the answer option D?

Answers

Answer:

D

Step-by-step explanation:

graph it

Other Questions
A graph of 2 functions is shown below. graph of function f of x equals negative 11 by 3 multiplied by x plus 11 by 3 and graph of function g of x equals x cubed plus 2 multiplied by x squared minus x minus 2 Which of the following is a solution for f(x) = g(x)? (2 points) x = 2 x = 1 x = 0 x = 1 The system of equations shown below is graphed on a coordinate grid:3y + x = 42y x = 6Which statement is true about the coordinates of the point that is the solution to the system of equations?A. It is (2, 2) and lies on both lines.B. It is (5, 3) and lies on both lines.C. It is (5, 3) and does not lie on either line.D. It is (2, 2) and does not lie on either line.Please help asap!! WILL GIVE BRAINlIEST!! tysssm if u help!!! What is the V7 chord for the scale shown? A7 D7 E7 G7 What was the subject of Withencroft's sketch? The population, P(t), in millions, of a country, in year t, is given by the formula P(t) = 24 + 0.4t. What are the values of the population for t = 10, 20,and 30? Jika Kita ingin memasuki gua yang dalam ,Kita digalakkan untuk membawa obor api selain lampu suluh sebagai penunjuk kehadiran oksigen.Apakah kelebihan menggunakan obor di dalam gua? 10 sentences about post box Your merchant fee is 3%. Yourcustomers paid you $2,200,000through credit cards. What feemust you pay the credit cardcompany?$ [?] Describe the system of equations How many solutions does this system have. Jeanette wants to raise $3,200 in a marathon fundraiser. Her sponsers will donate$35 for each (whole) kilometer she runs this summer.The minimum amount Jeanette will have to run to reach her goal of $3, 200 iskilometers. Which is the main light-absorbing pigment for photosynthesis?O caroteneO chlorophyllO hemoglobinO anthocyanin PLEASE HELPPPPPPPP!!!!!!Part 2: Application and critical thinking (20 points) 7) Select a supreme court case that has been particularly important in protecting civil liberties in the United States. In a three paragraph essay, describe the case and the ruling, then explain how the case demonstrates why it is important that the supreme court is:. 1. Protected from the political or public pressure (since justices are appointed rather than elected and serve lifetime terms). 2. Able to change its interpretations of the law over time. 4. He .. books for children for years.A. collected B. has collected C. collects D. will collect What is the molecular geometry of CIO3F as predicted by the VSEPR model?Multiple Choicetrigonal pyramidalsquare planarsquare pyramidaltetrahedraloctahedral Simplify this expression.Can anyone help pls What was the bush doctrine ? Ibrahim likes to run a loop around the park near his house that is mile long. There is a water fountain way around the loop. Ibrahim stopped to get a drink of water at the water fountain. How far did Ibrahim run? Simplifyx * x^5 / x^2 * x what do the authors of the decameron and the manifesto of the revolting cardinals belive about the nature of man There is a bag with 50 popsicles inside. 5 are red, 15 are orange, 12 are blue, 8 areyellow and 10 are purple. If you were tograb one popsicle from the bag, what isthe probability that it is red or not orange?P(red or not orange)